Как включается в цепь вольтметр: Как включается в цепь вольтметр и что он измеряет?

Содержание

Электрическое напряжение. Вольтметр — урок. Физика, 8 класс.

Пробовали ли вы когда-нибудь надувать воздушные шарики на время? Один надувает быстро, а другой за это же время надувает гораздо меньше. Без сомнения, первый совершает большую работу, чем второй.

 

 

Рис. \(1\). Надувание шара

 

С источниками напряжения происходит точно так же. Чтобы обеспечить движение частиц в проводнике, надо совершить работу. И эту работу совершает источник. Работу источника характеризует напряжение. Чем оно больше, тем большую работу совершает источник, тем ярче будет гореть лампочка в цепи (при других одинаковых условиях).

 

 

Рис. \(2\). Лампа в цепи

 

Напряжение равно отношению работы электрического поля по перемещению заряда
к величине перемещаемого заряда на участке цепи.

U=Aq, где \(U\) — напряжение, \(A\) — работа электрического поля, \(q\) — заряд.

 

Обрати внимание!

Единица измерения напряжения в системе СИ — [\(U\)] = \(1\) B (вольт).

\(1\) вольт равен электрическому напряжению на участке цепи, где при протекании заряда, равного \(1\) Кл, совершается работа, равная \(1\) Дж: \(1\) В \(= 1\) Дж/1 Кл.

Все видели надпись на домашних бытовых приборах «\(220\) В». Она означает, что на участке цепи совершается работа \(220\) Дж по перемещению заряда \(1\) Кл.

 

Кроме вольта, применяют дольные и кратные ему единицы — милливольт и киловольт.

\(1\) мВ \(= 0,001\) В, \(1\) кВ \(= 1000\) В или \(1\) В \(= 1000\) мВ, \(1\) В \(= 0,001\) кВ.

Для измерения напряжения используют прибор, который называется вольтметр.

Обозначаются все вольтметры латинской буквой \(V\), которая наносится на циферблат приборов и используется в схематическом изображении прибора.

 

 

Рис. \(3\). Обозначение вольтметра

 

В школьных условиях используются вольтметры, изображённые на рисунке:

 

 

 

Рис. \(4\). Вольтметры

 

Основными элементами вольтметра являются корпус, шкала, стрелка и клеммы. Клеммы обычно подписаны плюсом или минусом и для наглядности выделены разными цветами: красный — плюс, черный (синий) — минус. Сделано это с той целью, чтобы заведомо правильно подключать клеммы прибора к соответствующим проводам, подключённым к источнику.

 

Обрати внимание!

В отличие от амперметра, который включается в разрыв цепи последовательно, вольтметр включается в цепь параллельно.

 

Рис. \(5\). Электроцепь с подключенным вольтметром и амперметром

 

Включая вольтметр в цепь постоянного тока, необходимо соблюдать полярность.

 

Сборку электрической цепи лучше начинать со всех элементов, кроме вольтметра, а его уже подключать в самом конце.

Вольтметры делятся на приборы постоянного тока и переменного тока.

Если прибор предназначен для цепей переменного тока, то на циферблате принято изображать волнистую линию. Если прибор предназначен для цепей постоянного тока, то линия будет прямой.

 

Таблица \(1\). Вольтметры

 

Рис. \(6\). Вольтметр постоянного тока

Рис. \(7\). Вольтметр переменного тока

 

Можно обратить внимание на клеммы прибора. Если указана полярность («\(+\)» и «\(-\)»), то это прибор для измерения постоянного напряжения.


Иногда используют буквы \(AC/DC\). В переводе с английского \(AC\) (alternating current) — переменный ток, а \(DC\) (direct current) — постоянный ток.
В цепь переменного тока включается вольтметр для измерения переменного тока. Он полярности не имеет.

 

 

Рис. \(8\). Электроцепь с переменным источником тока

 

Обрати внимание!

Для измерения напряжения можно использовать и мультиметр.

Перед измерением необходимо прочитать инструкцию, чтобы правильно подключить прибор.

 

 

Рис. \(9\). Мультиметр

 

Следует помнить, что высокое напряжение опасно.

Что будет с человеком, который окажется рядом с упавшим оголённым кабелем, находящимся под высоким напряжением?

Так как земля является проводником электрического тока, вокруг упавшего оголённого кабеля, находящегося под напряжением, может возникнуть опасное для человека шаговое напряжение.

 

При попадании под шаговое напряжение даже небольшого значения возникают непроизвольные судорожные сокращения мышц ног. Обычно человеку удаётся в такой ситуации своевременно выйти из опасной зоны.

 

Обрати внимание!

Однако нельзя выбегать оттуда огромными шагами, шаговое напряжение при этом только увеличится! Выходить надо обязательно быстро, но очень мелкими шагами или скачками на одной ноге!

Существует много знаков, предупреждающих о высоком напряжении. Вот некоторые из них.

 

   

 

Рис. \(10\). Предупреждающие об опасности знаки

 

Безопасным напряжением для человека считается напряжение \(42\) В в нормальных условиях и \(12\) В в условиях с повышенной опасностью (сырость, высокая температура, металлические полы и др.).

Источники:

Рис. 5. Электроцепь с подключенным вольтметром и амперметром. © ЯКласс.
Рис. 8. Электроцепь с переменным источником тока. © ЯКласс.

Вольтметры — Включение — Схема

Вычислите относительную погрешность показаний вольтметра, включенного по схеме, приведенной на рис. 11, которая получается, если предположить, что вольтметр имеет бесконечно большое сопротивление и не вносит искажений в измеряемую цепь. Классифицируйте погрешность измерения для этой задачи.  [c.44]

Ремонт стартера. Если есть сомнения в эффективности работы стартера, необходимо проверить его на стенде с включением по схеме согласно рис. 161. При проверке используются хорошо заряженная аккумуляторная батарея, вольтметр постоянного тока со шкалой 0—30 В, амперметр с шунтом до 100 А, тахометр и динамометр. Температура стартера должна быть 25 3 °С, а щетки хорошо притерты к коллектору.  [c.223]


Измерения величины изменения индукции ДВу производятся с помощью амплитудного вольтметра У , включенного на измерительную обмотку через интегратор. К амплитудному вольтметру в описываемой схеме предъявляются высокие требования, связанные с тем, что при небольшой средней величине измеряемых э. д. с. пики нанряжения в обмотке образца могут быть в десятки раз большими по сравнению со средней их величиной.
Поэтому усилитель вольтметра должен обладать большой широкополосностью.  [c.289]

Принципиальная схема такого прибора приведена на рис. 97. Он представляет собой ползунковый реостат сопротивлением 100— 200 ом (ток 1 — 0,75 а), включенный по схеме потенциометра, с подвижного контакта которого снимается требуемое напряжение, контролируемое вольтметром.  

[c.127]

Рис. 2.2.4. Зависимость показаний вольтметра термоанемометра от скорости потока при включении по схеме с постоянной температурой датчика
Проволочный датчик термоанемометра закреплялся на координатнике и устанавливался в рабочей части так, что продольная ось датчика совпадала с осью трубы. После включения электрической схемы термоанемометра и балансировки измерительного канала вольтметром постоянного тока было измерено начальное напряжение 11о=3 в.  [c.147]

Через выпрямитель усиленного электродренажа, включенный между трубопроводом и рельсом, при малом вторичном напряжении трансформатора могут течь блуждающие токи от трубопровода к рельсу, если отрицательное напряжение трубопровод — рельс больше первоначального напряжения холостого хода этого выпрямителя.

Такое состояние обнаруживается по отклонению вольтметра защитной установки в противоположную сторону, причем через установку может протекать очень большой ток. Перегрузка установки в таком случае предотвращается соответствующей автоматической схемой. Реле максимального тока вызывает срабатывание другого реле, которое разъединяет выходную цепь тока трубопровод — защитная установка — рельс и при необходимости обеспечивает прямое соединение трубопровод — рельс. При помощи настраиваемого часового механизма разъединительное реле включается снова. В итоге станция продолжает работать. Число произошедших отключений указывается на счетчике. Это позволяет контролировать работу станции и дает представление о частоте отключений и тем самым о неполадках в работе электрифицированной железной дороги.  
[c.227]


НИИ 4 автоматического ключа реверсивного двигателя 5 для выключения двигателя при поступлении на вход одновибратора полезного сигнала или помех реле времени 6 для включения звукового или светового сигнала 7 импульсного вольтметра 12 для измерения напряжения сигналов до ограничения и после него, что позволяет правильно настроить сигнализирующее устройство по коэффициенту оптического отражения поверхности образца в начале испытания.
Кроме того, в электрическую схему устройства входят каскад питания устройства сигнализации 8, лампа накаливания 9 со стабилизатором 10, реверсивный двигатель поискового механизма 11 и каскад питания поискового механизма 13. Отраженный поверхностью вращающегося образца свет  [c.186]

Так как при сборке приборов используется недостаточно стабилизированный манганин, то в процессе работы показания приборов могут измениться. Особое значение это имеет в тех случаях, когда применяемое в приборе манганиновое сопротивление, включенное в последовательной схеме, велико по сравнению с общим сопротивлением прибора. В некоторых вольтметрах, где общее сопротивление прибора равно 3255 ом, сопротивление манганина 3250 ом, нестабильность вольтметров больше на 30 нестабильности аналогичной конструкции амперметров.  

[c.105]

Вольтамперные характеристики фотоэлемента 364 Вольтметры — Включение — Схема 373 Вольтодобавочные машины — Определение 378  [c. 535]

Электрическая схема включения котла (рис. 18,а) имеет автоматический выключатель, служащий для защиты от перегрузок и коротких замыканий контактор для коммутации цепи подключения электродного котла трансформаторы тока и амперметры, предназначенные для контроля токов нагрузки электродного котла вольтметры для контроля напряжения питания.  

[c.89]

Измерение тока и напряжения. Схемы непосредственного включения и через измерительные трансформаторы тока и напряжения. Амперметры и вольтметры различных систем.  [c.326]

С изменением диаметра обрабатываемой детали происходит перемещение измерительного штока 1, а следовательно, поворот рычага с заслонкой, перекрывающей щель в диафрагме и изменяющей интенсивность альфа-излучения, что регистрируется счетчиком Гейгера, включенным в электрическую схему датчика (фиг. 207). Интенсивность излучения пропорциональна напряжению на нагрузочном сопротивлении 1, измеряемым катодным вольтметром, работающим яа двойном триоде 6Н8, между катодами которого включен микроамперметр 2 и реле 3.

Микроамперметр 2 служит для настройки датчика и визуального наблюдения за изменением диаметра обрабатываемой детали. При достижении ею требуемого размера срабатывает реле 5, включающее сигнальную лампочку 4 и подающее команду на останов станка.  [c.207]

Для проверки регулятора напряжения необходимо иметь следующие приборы вольтметр постоянного тока со щкалой до 20—30 б и ценой деления 0,1— 0,2 в амперметр постоянного тока со щкалой до 30 а (шкала двусторонняя с нулевым делением посредине) и ценой деления I а. Схема включения приборов для проверки регулятора напряжения приведена на рис. 167.  [c.252]

Приборы и приспособления контрольный стенд или (при отсутствии стенда) тиски амперметр с шунтом вольтметр тахометр. Электрическая схема включения стартера для проверки приведена на рис. 171.  [c.257]

На рис. 31 показана схема включения приборов и вспомогательных устройств при общей проверке и регулировке установки.

Вольтметр постоянного тока Fi со шкалой О—3 в служит для контроля сигнального напряжения на входе блока управления. Амперметр Ai со шкалой О—1 а необходим для измерения тока в цепи  [c.81]


Проверяя цепь для определения места обрыва, следует помнить, что у многих кранов часть цепи работает на переменном токе, а часть — на постоянном (например, цепь реле КЗ на рис. 184). При проверке цепи постоянного тока зажимы вольтметра (лампы) подключают к источнику постоянного тока, а при проверке цепи переменного тока — к фазе переменного тока. Во время работы следует обязательно пользоваться электрическими схемами, так как ошибочное включение лампы в фазу переменного тока при проверке цепи, работающей на постоянном токе, может привести к повреждению выпрямительных устройств.  
[c.539]

Проверка и регулировка дополнительного реле. Схема соединений при проверке дополнительного реле приведена на рис. 28. После замыкания цепи включателем 9 плавно повышают напряжение реостатом 8, пока не загорится лампа 7, показывая, что контакты реле замкнулись. В этот момент вольтметр показывает напряжение включения реле. Затем реостатом плавно понижают напряжение до того момента, когда лампочка погаснет, свидетельствуя о размыкании контактов. При этом показание вольтметра соответствует напряжению выключения реле. Результаты проверки должны соответствовать значениям, указанным в табл. 9.  

[c.59]

Калориметр выполнен с двойными стенками, между которыми циркулирует охлаждающая вода. Значительный расход воды обеспечивает постоянство температуры внутренней поверхности калориметра, которая является тепловоспринимающей. Внутренний диаметр калориметра значительно больше диаметра проволоки. Поверхность проволоки не только излучает энергию, но и участвует в процессах конвективной теплоотдачи и теплопроводности. Однако после вакуумирования при остаточном давлении воздуха внутри калориметра порядка 10 мм рт.
ст. передача теплоты путем конвекции и теплопроводности становится пренебрежимо малой, и проволока передает теплоту станкам калориметра только излучением. Тепловой поток определяется по падению напряжения на измерительном участке и силе тока в нем. Падение напряжения измеряется цифровым вольтметром Ф219 через делитель напряжения. Силу электрического тока, проходящего через проволоку, определяют с помощью образцового сопротивления (У н = 0,05 Ом), включенного в схему. Сила тока изменяется в пределах 1—3 А. Падение напряжения на образцовом сопротивлении измеряется с помощью того же цифрового вольтметра. На измерительном участке температура проволоки практически постоянна по длине. Эта температура определяется П0 зависимости электрического сопротивления проволоки от температуры. Такой измерительный преобразователь температуры носит название термометра сопротивления (см. п. 3.1.2). Зависимость электрического сопротивления исследуемого тела от температуры определяется предварительными опытами.[c.189]

Сопротивление контактов в местах присоединения отрицательных питающих линий измеряется вольтметром с внутренним сопротивлением не менее 10 ком на 1 в и амперметром, включенным по схеме рис. 15. Сопротивление контакта определяется как разность между сопротивлением, вычисленным по показанию приборов, и расчетным сопротивлением соответствующего проводника, соединяющего отрицательную питающута линию с рельсовой нитью.  [c.98]

Обмотка реле R не запитана, реле сработать не может, и его нормально замкнутый контакт R, включенный в схему последовательно с сигнальной лампой Останов (см. поз.5), обязательно замкнут. Но лампа ни что иное, как обычное сопротивление, поэтому потенциал ОВ постоянно присутствует в точке 4 и вольтметр покажет 24В независимо от положения рубильника М/А.  [c.303]

Без включения в схему вольтметров, имеющих малое внутреннее сопротивление, падения напряжения на каждой половине реостата будут одинаковы Ujl В, т. е. 90 В, поэтому погреишость измерения первым вольтметром следует оценить как +10%, а второго — 10 %.  [c.95]

Ваттметр, включенный в схему, измеряет сумму полных потерь в образце, помещенном в аппарат, и мощностей, потребляемых обмоткой вольтметра и вольтметровой обмоткой ваттметра.  [c.212]

Груз О, укрепленный на пружине L, отклоняется под влиянием действующей на него силы инерции (перегрузки). Движение груза связано посредством тяги т с рычагом Kf вращающимся на оси О. Рычаг К имеет на своем конце контактное асолесико i, катающееся по реостату. Между одним концом А реостата и подвижным рычагом К включен вольтметр (не показан на схеме). Через реостат пропущен ток от 6—8-вольтной батареи, и вольтметр, таким образом, служит потенцио-  [c.32]

Проведение эксперимента на модели. Решающая схема (рис. 5.5) представлена на демонстрационной панели лабораторного стенда. В узлах схемы установлены электрические гнезда, с которых снимаются значения выходных величин решающих элементов схемы. Для регистрации решения используются электронно-лучевой индикатор (ИЭЛ) И-б я цифровой вольтметр типа Щ1312. Порядок подключения этих приборов к схеме указан ниже. На схеме и демонстрационной панели показаны два функциональных преобразователя, реализующих зависимости i(t) для АЬОз и 2гОг. Включение их в схему осуществляется одновременным переводом тумблеров 5 и б соответственно в верхнее (для АЬОз) или нижнее (для ZrOj) положение.  [c.212]


Для использования установки при исследованиях зависимости вязкости жидкостей от температуры и давления был разработан и изготовлен вариант капельной и защитной трубок, в котором защитная трубка выполнена из стали 1Х18Н9Т, а регистрация времени падения ртути осуществляется с помощью платиновых контактов. Для этого Б капельную трубку впаиваются платиновые контакты, которые при замыкании ртутью обеспечивают соответствующий импульс. Однако, как показали наладочные опыты на МИПД, вокруг ртутного столбика образовывается изолирующая пленка, которая вызывает ненадежное включение сигнального устройства. В связи с этим отсчет времени в вискозиметре производился или визуально, или с помощью контура электромагнитных колебаний. Схема колебательного контура (рис. 3-33) состоит из трех индуктивных катушек, двух конденсаторов постоянной емкости (50 и 240 пф), стандартного генератора звуковых сигналов (СГС-1) и катодного вольтметра ВДУ-2. Индуктивные катушки намотаны на капельную трубку вискозиметра. Катушки примерно одинаковы, а их длина равна высоте ртутного столбика.  [c.169]

Принципиальная схема такого прибора приведена выше (см. рис. 4). Контролируемое изделие 1 просвечииается узким пучком 7-излу-чення F . Вторым пучком -/-излучеипя F. просвечивается эталонное изделие 2. Изделия располагаются и перемещаются так, что потоки -излучения всегда попадают на участки изделий, которые должны иметь одинаковую толщину. При отклонении внешних размеров контролируемого изделия и наличии в нем внутренних дефектов на фосфоры и Ф, попадают разные П0Т01Ш -у-излучения. Этот факт обнаруживается по отклонению стрелки вольтметра, включенного на выходе синхронного детектора.  [c.132]

Питание мотора / осуществляется по схеме Леонарда от специального генератора постоянного тока ДУ/ Г (динамо, управляющая работой головки), объединённого с мотором трёхфазного тока во вспомогательный моторгене-раторный агрегат. Независимая обмотка возбуждения генератора питается через ку-проксные выпрямители НКС-2 от напряжения на дуге. Возбуждение мотора I также зависит от напряжения на дуге. Такая схема включения обеспечивает плавное изменение скорости подачи электродной проволоки в зависимости от напряжения дуги. Мотор 2 — асинхронный, с постоянным числом оборотов — служит для возбуждения дуги в начале сварки и создания необходимого числа оборотов на выходном валу диференциала. Контроль за режимом сварки осуществляется по амперметру А и вольтметру V.[c.339]

При расчете количества тепла, подводимого от калориметрического нагре1вателя, следует учесть, что при схеме включения измерительных приборов, показанной на рис. 9-3, амперметр измеряет суммарную силу тока, протекающего в цепи нагревателя и вольтметра. Поэтому сила тока, протекающего в цепи нагревателя, определяется как  [c.266]

Прибор имеет настольное оформление. Внутри его корпуса, на двух выдвижных панелях, смонтированы узлы электроизмерительной схемы, регулятор напряжения питания нагревателя и распределительная система водяного охлаждения. На лицевую панель прибора вынесены рукоятки управления, кнопки включения и выключения прибора, тумблер включения нагревателя, переключатели масштаба записи сигналов термопар и режима работы, контрольный манометр системы охлаждения и контрольные амперметр и вольтметр нагревательной цепи. В комплект прибора входит шеститочечный электронный потенциометр типа ЭПП-09.  [c.63]

Корректирование работы регулятора. Для нормальной работы трансформатора необходимо, чтобы токи, проходящие через каждый из двух включенных встречно-параллельно игнитронов, были равны. При нарушении этого условия в токе, проходящем через обмотку высокого напряжения трансформатора Т, появляется постоянная составляющая, вызывающая подмагни-чивание трансформатора и дополнительный нагрев его. Для наблюдения за постоянной составляющей игнитронный регулятор имеет индикатор постоянной составляющей ИПС, представляющий вольтметр с нулем посередине для наблюдения за этой составляющей в обоих направлениях. Устранение постоянной составляющей, появляющейся вследствие разброса параметров ламп или нарушения симметрии схемы, производится изменением сопротивления Ru (рукоятка с надписью Корректор ),  [c.157]

Ответ. Предпочтительней схема на рис. 40, б внутреннее сопротивление вольтметра R-a должно быть возможно большим по сравнению с сопротивлением участка пепи, на котором производится измерение, чтобы избежать погрешности из-за уменьшения этого сопротивления при включении прибора. Включив прибор через конденсатор С, можно обеспечить режим, когда реактивное сопротивление Хс будет значительно меньше При С = 0,1 мкФ Хс = Ю /2я/С 1500 Ом, а = 2S0000 Ом, т. е. X [c.122]

Ваттметрический метод определения полных потерь на гистерезис и вихревые токи [36]. Ваттметрический метод основан на измерении потерь мощности в трансформаторе с разомкнутой вторичной цепью (т. е. не потребляющий мощности), причем в качестве сердечника трансформатора используется испытуемый материала (аппарат Эпштейна). Принципиальная схема установки представлена на рис. 17.68. В четыре секции трансформатора П], Пг набирается образец из пластин, которые образуют магнитную цепь. В цепь первичной намагничивающей катушки щ включен амперметр А и токовая обмотка ваттметра в цепь вторичной обмотки трансформатора включены вольтметр V и обмотка напряжения ваттметра —1 2. Полные потери на гистерезис и вихревые токи Рт. в равны Р . в = ( — E 2lR2]wl w2, где Р — показания ваттметра  [c. 317]

Крутящий момент преобразуется в пропорциональный ему электрический сигнал. Тензодатчики включены в уравновешенный мост, питаемый от источника постоянного напряжения 15 в. К мосту может подключаться схема калибровки тен-зодатчиков с вольтметром и источником питания. Электрический хиг-нал от тензодатчиков подается на клеммы X самописца. На клеммы У самописца подается напряжение от тахогенератора постоянного тока, откалиброванного с точностью 0,5%. Тахогенератор дает напряжение 25 в при 1000 об1мин (режим холостого хода). Для снижения подаваемого на клеммы У самописца выходного сигнала до 10 мв включен набор сопротивлений. Величина тока в цепи контролируется амперметром.  [c.195]

МИ электрооптическими кристаллами типа u l, ZnS или НМТ 65, 66] или одноосными кристаллами типа KDP и ADP в продольной конфигурации (свет распространяется вдоль оптической оси с). Тот факт, что в настоящее время нет кристаллов достаточно высокого оптического качества, можно рассматривать как временный [70]. Модуляторы чаще изготавливают из более доступных материалов KDP и ADP в поперечной конфигурации [67, 68]. Но поскольку при таком методе измерения через кристалл должны проходить две (перпендикулярно) поляризованные компоненты, даже если модулятор предназначен для фазовой модуляции линейно поляризованного излучения, мы сталкиваемся с проблемой естественного двойного лучепреломления и нестабильных оптических искал ений, которые ограничивают применимость поперечной конфигурации с кристаллами KDP и ADP для модуляции по интенсивности [69]. В схеме, изображенной на фиг. 9,4, за модулятором стоит фотоумножитель, на выходе которого включен чувствительный вольтметр постоянного тока. Поскольку измеряются только средние интенсивности, фотоумножитель может иметь большую постоянную времени.  [c.489]

Проверяют исправность цепи присоединения клеммы 102 к каркасу панели управления. Для этого при включенном вводном рубильнике присоединяют указатель напряжения (вольтметр) к очищенному от краски и грязи каркасу панели управления, касаясь вторым щупом предохранителя цепи управления лифтом, убеждаются, что цепь присоединения клеммы 102 к каркасу панели управления исправна (лампа указателя напряжения светится, стрелка вольтметра отклоняется). Затем проверяют надежность присоединения каркаса панели управления к зануляюшей магистрали. Работа лифта должна быть приостановлена, если при проведении технического осмотра обнаружены, иибо неисправность цепи зануления клеммы 102 схемы управления лифтом, либо отсутствие зануления каркасов электрооборудования и электроаппаратов, а также зануления вторичных обмоток понижающих трансформаторов.  [c.272]


Напряжение от сети через блокировочные контакты и предохранители подводится к регулировочному автотрансформатору Т1, служащему для плавного изменения напряжения, и к трансформатору накала кенотрона Т2 (рис. 29.52). Включение высокого напряжения осуществляется нажатием кнопки S1 автоматического выключателя, имеющего три обмотки две из них соединены последовательно (причем одна шунтируется переключателем защиты S2). Разомкнутое положение этого переключателя соответствует чувствительной защите автомат срабатывает при пробое на стороне переменного тока и остается включенным, если ток в цепи выпрямленного напряжения не превосходит 5 мА. Когда переключатель 52 замкнут, осуществляется грубая защита автомат не срабатывает при коротком замыкании на высокой стороне и остается включенным, если мощность на стороне высокого напряжения при 50 кВ пе превосходит 2 кВ-А такой режим должен длиться не более 1 мин. Измерение напряжения на образце производится вольтметром kV класса 1,5 на стороне низкого напряжения, проградуированным в киловольтах. Конденсаторы С служат для защиты от перенапряжений первичной обмотки. При синусоидальной форме кривой питающего напряжения вторичное напряжение высоковольтного трансформатора в режиме холостого хода не отличается от синусоидального более чем на 5 %. Резистор R служит для защиты трансформатора и кенотрона от перегрузки при пробое образца. В установке имеется сосуд с электродами для стандартного испытания жидких материалов. Испытания на постоянном токе производят при помощи схемы однополупериодного выпрямления, для получения которой йспо.тьзу-ется кенотрон Л на образец подается постоянное напряжение отрицательной полярности. Если необходимо измерять ток утечки, то для этой цели используют микроамперметр в анодной цепи. Защита мнкроамперметра от перегрузок осуществляется при помощи разрядника Р, шунтирующего конденсатор, и сопротивле-  [c.394]

В нижней части рисунка показана последовательность преобразования подводимой энергии напряжением /л промышленной частоты 50 Гц в частоту 10 000 Гц в машинном преобразователе. Высокое напряжение С/1 с помощью понижающего трансформатора трансформируется в напряжение С/г, не превышающее нескольких десятков вольт. Контроль электрических параметров процесса нагрева детали осуществляется по приборам, схема включения которых изображена на рис. 61. В схему включаются пять приборов вольтметр В, амперметр А, киловаттметр КВ для измерения соответственно напряжения, тока и мощности генератора фазометр Ф для измерения коэффициента мощности на-  [c.108]

К — электрод, поляризуемый катодно А — электрод, поляризуемый анодно НЭ — каломельные полуэлементы КВ — клеммы для присоединения катодного вольтметра НЭ — нормальный элемент Вестона М — мешалка с гидравлическим затвором Г — шлифы с кранами для ввода газов П — отбор проб электролита Пх — переключатель для включения катодного вольтмет ра в цепь катода пли анода и Я-, — переключатели для включения в измерительную схему элемента Вестона (включается при э. д. с. > 1 в)  [c.137]


вольтметр — это… Что такое вольтметр?

  • вольтметр — вольтметр …   Орфографический словарь-справочник

  • вольтметр — сущ., кол во синонимов: 13 • ампервольтваттметр (4) • вольтамперметр (6) • …   Словарь синонимов

  • вольтметр — а, м. voltmètre, voltamètre. Вольтаметр. Электроизмерительный прибор для измерения напряжения между двумя точками электрической цепи. СИС 1954. См. Вольтаметр. Лекс. Брокг.: вольтметр; САН 1891: вольтме/тр …   Исторический словарь галлицизмов русского языка

  • ВОЛЬТМЕТР — ВОЛЬТМЕТР, прибор для измерения электродвижущей силы или электрического напряжения в вольтах (В). В электрическую цепь включается параллельно нагрузке. Шкалу вольтметра часто градуируют в кратных и дольных единицах от Вольта (мкВ, мВ, кВ) …   Современная энциклопедия

  • ВОЛЬТМЕТР — прибор для измерения электродвижущей силы или напряжения (в мкВ, мВ, В, кВ) в электрических цепях; включается параллельно нагрузке …   Большой Энциклопедический словарь

  • ВОЛЬТМЕТР — ВОЛЬТМЕТР, прибор для измерения напряжения (РАЗНОСТИ ПОТЕНЦИАЛОВ) между двумя точками электрической схемы. Вольтметры должны подключаться параллельно элементам, напряжение на которых нужно измерить. Имеет высокое внутреннее СОПРОТИВЛЕНИЕ (по… …   Научно-технический энциклопедический словарь

  • ВОЛЬТМЕТР — ВОЛЬТМЕТР, вольтметра, муж. (физ.). Прибор для определения электрического напряжения, вольтажа. Толковый словарь Ушакова. Д.Н. Ушаков. 1935 1940 …   Толковый словарь Ушакова

  • ВОЛЬТМЕТР — ВОЛЬТМЕТР, а, муж. Прибор для измерения электродвижущей силы и напряжения в электрической цепи. Толковый словарь Ожегова. С.И. Ожегов, Н.Ю. Шведова. 1949 1992 …   Толковый словарь Ожегова

  • ВОЛЬТМЕТР — (от вольт и греч. metreo измеряю), прибор для измерения напряжения в электрич. цепях. В. включается параллельно участку цепи, на к ром измеряется напряжение. Для уменьшения влияния включённого В. на режим цепи он должен обладать большим входным… …   Физическая энциклопедия

  • ВОЛЬТМЕТР — (Voltmeter) прибор для измерения электрического напряжения. Самойлов К. И. Морской словарь. М. Л.: Государственное Военно морское Издательство НКВМФ Союза ССР, 1941 …   Морской словарь

  • ВОЛЬТМЕТР — прибор для измерения электр. напряжения. По принципу действия В. распространенных типов аналогичны амперметрам и отличаются от них: а) величиной электр. сопротивления катушек (в В. они имеют большое сопротивление, а в амперметрах малое; В.,… …   Технический железнодорожный словарь

  • Измерение тока и напряжения. Вольтметр и амперметр.

    Приветствую всех читателей на нашем сайте и сегодня в рамках курса «Основы электроники» мы будем изучать основные способы измерения силы тока, напряжения и других параметров электрических цепей. Естественно, без внимания не останутся и основные измерительные приборы, такие как вольтметр и амперметр.

    Измерение тока. Амперметр.

    И начнем мы с измерения тока. Прибор, используемый для этих целей, называется амперметр и в цепь он включается последовательно. Рассмотрим небольшой примерчик:

    Как видите, здесь источник питания подключен напрямую к резистору. Кроме того, в цепи присутствует амперметр, включенный последовательно с резистором. По закону Ома сила тока в данной цепи должна быть равна:

    I = \frac{U}{R} = \frac{12}{100} = 0.12

    Получили величину, равную 0.12 А, что в точности совпадает с практическим результатом, который демонстрирует амперметр в цепи 🙂

    Важным параметром этого прибора является его внутреннее сопротивление r_А. Почему это так важно? Смотрите сами — при отсутствии амперметра ток определяется по закону Ома, как мы и рассчитывали чуть выше. Но при наличии амперметра в цепи ток изменится, поскольку изменится сопротивление, и мы получим следующее значение:

    I = \frac{U}{R_1+r_А}

    Если бы амперметр был абсолютно идеальным, и его сопротивление равнялось нулю, то он бы не оказал никакого влияния на работу электрической цепи, параметры которой необходимо измерить, но на практике все не совсем так, и сопротивление прибора не равно 0. Конечно, сопротивление амперметра достаточно мало (поскольку производители стремятся максимально его уменьшить), поэтому во многих примерах и задачах им пренебрегают, но не стоит забывать, что оно все-таки и есть и оно ненулевое.

    При разговоре об измерении силы тока невозможно не упомянуть о способе, который позволяет расширить пределы, в которых может работать амперметр. Этот метод заключается в том, что параллельно амперметру включается шунт (резистор), имеющий определенное сопротивление:

    R = \frac{r_А}{n\medspace-\medspace 1}

    В этой формуле n — это коэффициент шунтирования — число, которое показывает во сколько раз будут увеличены пределы, в рамках которых амперметр может производить свои измерения. Возможно это все может показаться не совсем понятным и логичным, поэтому сейчас мы рассмотрим практический пример, который позволит во всем разобраться.

    Пусть максимальное значение, которое может измерить амперметр составляет 1 А. А схема, силу тока в которой нам нужно определить имеет следующий вид:

    Отличие от предыдущей схемы заключается в том, что напряжение источника питания на этой схеме в 100 раз больше, соответственно, и ток в цепи станет больше и будет равен 12 А. Из-за ограничения на максимальное значение измеряемого тока напрямую использовать наш амперметр мы не сможем. Так вот для таких задач и нужно использовать дополнительный шунт:

    В данной задаче нам необходимо измерить ток I. Мы предполагаем, что его значение превысит максимально допустимую величину для используемого амперметра, поэтому добавляем в схему еще один элемент, который будет выполнять роль шунта. Пусть мы хотим увеличить пределы измерения амперметра в 25 раз, это значит, что прибор будет показывать значение, которое в 25 раз меньше, чем величина измеряемого тока. Нам останется только умножить показания прибора на известное нам число и мы получим нужное нам значение. Для реализации нашей задумки мы должны поставить шунт параллельно амперметру, причем сопротивление его должно быть равно значению, которое мы определяем по формуле:

    R = \frac{r_А}{n\medspace-\medspace 1}

    В данном случае n = 25, но мы проведем все расчеты в общем виде, чтобы показать, что величины могут быть абсолютно любыми, принцип шунтирования будет работать одинаково.

    Итак, поскольку напряжения на шунте и на амперметре равны, мы можем записать первое уравнение:

    I_А\medspace r_А = I_R\medspace R

    Выразим ток шунта через ток амперметра:

    I_R = I_А\medspace \frac{r_А}{R}

    Измеряемый ток равен:

    I = I_R + I_А

    Подставим в это уравнение предыдущее выражение для тока шунта:

    I = I_А + I_А\medspace \frac{r_А}{R}

    Но сопротивление шунта нам также известно (R = \frac{r_А}{n\medspace-\medspace 1}). В итоге мы получаем:

    I = I_А\medspace (1 + \frac{r_А\medspace (n\medspace-\medspace 1)}{r_А}\enspace) = I_А\medspace n

    Вот мы и получили то, что и хотели. Значение, которое покажет амперметр в данной цепи будет в n раз меньше, чем сила тока, величину которой нам и нужно измерить 🙂

    С измерениями тока в цепи все понятно, давайте перейдем к следующему вопросу, а именно определению напряжения.

    Измерение напряжения.

    Вольтметр.

    Прибор, предназначенный для измерения напряжения называется вольтметр. И, в отличие от амперметра, в цепь он включается параллельно участку цепи, напряжение на котором необходимо определить. И, опять же, в противоположность идеальному амперметру, имеющему нулевое сопротивление, сопротивление идеального вольтметра должно быть равно бесконечности. Давай разберемся с чем это связано:

    Если бы в цепи не было вольтметра, ток через резисторы был бы один и тот же и определялся по Закону Ома следующим образом:

    I_1 = I_2 = \frac{U}{R_1 + R_2} = \frac{30}{10 + 20} = 1

    Итак, величина тока составила бы 1 А, а соответственно напряжение на резисторе 2 было бы равно 20 В. С этим все понятно, а теперь мы хотим измерить это напряжение вольтметром и включаем его параллельно с R_2. Если бы сопротивление вольтметра было бы бесконечно большим, то через него просто не потек бы ток (I_B = 0), и прибор не оказал бы никакого воздействия на исходную цепь. Но поскольку r_В имеет конечную величину и не равно бесконечности, то через вольтметр потечет ток. В связи с этим напряжение на резисторе R_2 уже не будет таким, каким бы оно было при отсутствии измерительного прибора. Вот поэтому идеальным был бы такой вольтметр, через который не проходил бы ток.

    Как и в случае с амперметром, есть специальный метод, который позволяет увеличить пределы измерения напряжения для вольтметра. Для осуществления этого необходимо включить последовательно с прибором добавочное сопротивление, величина которого определяется по формуле:

    R_Д = r_В\medspace (n\medspace-\medspace 1)

    Это приведет к тому, что показания вольтметра будут в n раз меньше, чем значение измеряемого напряжения. По традиции давайте рассмотрим небольшой практический пример:

    Здесь мы добавили в цепь добавочное сопротивление R_3. Перед нами стоит задача измерить напряжение на резисторе R_2:\medspace U_2 = R_2\medspace I_2. Давайте определим, какой результат при таком включении выдаст нам вольтметр:

    U_2 = I_2\medspace R_2 = U_В + I_В\medspace R_3

    Подставим в эту формулу выражение для расчета сопротивления добавочного резистора:

    U_2 = U_В + I_В\medspace (r_В\medspace (n\medspace-\medspace 1)) = U_В + I_В\medspace r_В\medspace n\medspace-\medspace I_В\medspace r_В = U_В + U_В\medspace n\medspace-\medspace U_В = U_В\medspace n

    Таким образом: U_В = \frac{U_2}{n}. То есть показания вольтметра будут в n раз меньше, чем величина напряжения, которое мы измеряли. Так что, используя данный метод, возможно увеличить пределы измерения вольтметра!

    В завершении статьи пару слов об измерении сопротивления и мощности.

    Для решения обеих задач возможно совместное использование амперметра и вольтметра. В предыдущих статьях (про мощность и сопротивление) мы подробно останавливались на понятиях сопротивления и мощности и их связи с напряжением и сопротивлением, таким образом, зная ток и напряжение электрической цепи можно произвести расчет нужного нам параметра. Ну а кроме того есть специальные приборы, которые позволяют произвести измерения сопротивления участка цепи — омметр — и мощности — ваттметр.

    В общем-то, на этом, пожалуй, на сегодня закончим, следите за обновлениями и заходите к нам на сайт! До скорых встреч!

    Вольтметр. Зависимость силы тока от напряжения. Измерение напряжения на различных участках электрической цепи

    Тип урока: урок развивающего контроля и рефлексии.

    Используемые технологии: здоровьесбережения, информационно-коммуникационные, групповые, развития исследовательских навыков.

    Цель: закрепить и углубить теоретические и практические навыки учащихся по теме “Измерение напряжения на различных участках электрической цепи”.

    Формируемые УУД: предметные: научиться использовать приобретенные умения экспериментатора на практике; метапредметные: строить продуктивное взаимодействие со сверстниками; контролировать, корректировать и оценивать действия партнера; с достаточной полнотой и точностью выражать свои мысли в соответствии с задачами и условиями коммуникации; составлять план и последовательность действий; сравнивать результат и способ действий с эталоном с целью обнаружения отклонений и отличий; контролировать и оценивать процесс и результаты деятельности; формулировать выводы, адекватные полученным результатам; личностные: формирование самостоятельности в приобретении практических умений; усвоение правил поведения в школе; формирование бережного отношения к школьному оборудованию.

    Приборы и материалы: источник питания, лампочки, резисторы, ключи, амперметры, вольтметры, соединительные провода, электронное приложение к учебнику.

    Ход урока

    I. Организационный момент

    (Учитель и ученики приветствуют друг друга, выявляются отсутствующие. Учитель проводит инструктаж по работе с электрическими приборами.)

    II. Актуализация знаний. Проверка домашнего задания

    (Учитель проводит фронтальный опрос по вопросам и заданиям учебника, задает дополнительные вопросы. Три ученика записывают на доске решение домашних задач.)

    III. Изучение нового материала. Выполнение лабораторной работы

    Для измерения напряжения служит прибор, называемый вольтметром. В отличие от амперметра, который включается в разрыв электрической цепи, вольтметр подключается параллельно участку цепи, на котором измеряется напряжение.

    (Учитель демонстрирует учащимся анимационный ролик 91 “Включение вольтметра в цепь” из электронного приложения к учебнику. )

    Существует определенная зависимость силы тока в цепи от напряжения на источнике.

    Демонстрация. При увеличении количества источников питания (т. е. при увеличении напряжения) в электрической цепи, состоящей из резистора, ключа, амперметра и вольтметра, сила тока будет возрастать прямо пропорционально напряжению.

    (Учитель снимает несколько показаний с амперметра и вольтметра, и вместе с учащимися строит график зависимости силы тока от напряжения.)

    Вывод. Сила тока в проводнике прямо пропорциональна напряжению на его концах.

    (Ученики выполняют тренировочное задание № 92 из электронного приложения к учебнику. Учитель напоминает последовательность действий при измерении напряжения.)

    Последовательность действий при измерении напряжения

    1. Установить, для измерения какой величины используется данный прибор (если на шкале есть буква V — прибор используется для измерения напряжения).

    2. Установить, на какое максимальное напряжение рассчитан прибор.

    3. Установить, для какого тока (постоянного или переменного) можно использовать прибор.

    4. Определить цену деления прибора.

    5. Определить, какое значение напряжения показывает прибор.

    Во всех лабораторных опытах, где используется вольтметр, нужно сначала собрать цепь без него, а затем подключить вольтметр к тому участку, на котором измеряют напряжение. Вольтметр можно переключать от одного участка к другому, не разбирая остальные участки цепи.

    (Лабораторная работа выполняется согласно инструкции учебника. В ходе выполнения лабораторной работы ученики должны собрать последовательную электрическую цепь, состоящую из источника питания, ключа, двух реостатов, зарисовать ее в тетрадях, указав расположение вольтметра для трех случаев, и измерить напряжение сначала на каждом из реостатов в отдельности, а затем на двух сразу. Сравнить полученные значения напряжения.) (Ученики выполняют дополнительные задания.)

    1. Две лампы включены в электрические цепи, в которых сила тока равна, но, несмотря на это, одна из ламп горит менее ярко, чем другая. О чем свидетельствует этот факт? Какой вывод о напряжении на лампах можно сделать?

    2. При напряжении 100 В сила тока в резисторе равна 5 А. Какая сила тока будет в резисторе, если напряжение, поданное на него, уменьшить в два раза?

    IV. Подведение итогов

    (После выполнения лабораторной работы ученики самостоятельно делают вывод.)

    Вывод. При последовательном соединении проводников напряжение на них суммируется.

    V. Рефлексия

    (Ученики оценивают свою работу на уроке и качество усвоения материала, заполнив анкету.)

    Вопросы анкеты

    1. Вспомни и запиши тему урока.

    2. Какие термины, факты, закономерности ты усвоил(а) на уроке?

    3. Считаешь ли ты полезными, интересными полученные знания?

    4. Какую оценку за урок ты бы себе поставил(а)?

    Домашнее задание

    1. § 41,42 учебника, вопросы к параграфам.

    2. Выполнить упр. 26, 27 на с. 119, 121 учебника.

    метров — элементы схемы — содержание MCAT

    Прибор, используемый для измерения токов, называется амперметром, а для измерения разности потенциалов — вольтметром.

    Чтобы измерить ток в проводе, амперметр должен быть соединен в серии с другими элементами схемы. Важно, чтобы сопротивление амперметра было намного меньше других сопротивлений в цепи. В противном случае само присутствие измерителя изменит измеряемый ток.

    Чтобы найти напряжение , то есть разность потенциалов между любыми двумя точками в цепи, вольтметр подключается параллельно этим двум точкам. Важно, чтобы сопротивление вольтметра было намного больше, чем сопротивление любого элемента схемы, к которому подключен вольтметр. В противном случае измеритель изменяет измеряемую разность потенциалов.

    Часто одиночный измеритель — мультиметр — сконструирован таким образом, что с помощью переключателя он может использоваться как амперметр или вольтметр — и обычно также как омметр , предназначенный для измерения сопротивления любого элемента. подключен между его выводами.


    Ключевые точки

    • Амперметр измеряет ток в проводе, который должен быть включен в цепь последовательно.

    • Вольтметр измеряет разность потенциалов между двумя точками, которые необходимо подключить параллельно этим двум точкам.

    • Мультиметр — это устройство, которое может включать в себя амперметр, вольтметр и омметр.


    Ключевые термины

    ток : количество заряда, проходящего через поперечное сечение за период времени.

    напряжение : разность электрических потенциалов, выраженная в вольтах

    сопротивление : сопротивление является мерой противодействия протеканию тока в электрической цепи

    амперметр : прибор для измерения электрического тока в амперах

    серия: все компоненты в последовательной цепи соединены встык

    вольтметр: прибор, используемый для измерения разности электрических потенциалов между двумя точками в электрической цепи

    Омметр: Электрический прибор для измерения электрического сопротивления

    21.

    4 Вольтметры и амперметры постоянного тока — Физика колледжа, главы 1-17

    Сводка

    • Объясните, почему вольтметр нужно подключать параллельно цепи.
    • Нарисуйте схему, показывающую правильно подключенный амперметр в цепь.
    • Опишите, как гальванометр можно использовать как вольтметр или амперметр.
    • Найдите сопротивление, которое необходимо подключить последовательно с гальванометром, чтобы его можно было использовать в качестве вольтметра с заданными показаниями.
    • Объясните, почему измерение напряжения или тока в цепи никогда не может быть точным.

    Вольтметры измеряют напряжение, а амперметры измеряют ток. Некоторые измерители в автомобильных приборных панелях, цифровых камерах, сотовых телефонах и тюнерах-усилителях являются вольтметрами или амперметрами. (См. Рис. 1.) Внутренняя конструкция простейшего из этих счетчиков и то, как они подключены к системе, которую они контролируют, позволяют лучше понять применение последовательного и параллельного подключения.

    Рисунок 1. Датчики топлива и температуры (крайний правый и крайний левый, соответственно) в этом Volkswagen 1996 года представляют собой вольтметры, которые регистрируют выходное напряжение «передающих» устройств, которое, как мы надеемся, пропорционально количеству бензина в баке и температура двигателя.(Фото: Christian Giersing)

    Вольтметры подключаются параллельно к любому устройству, которое необходимо измерить. Параллельное соединение используется потому, что параллельные объекты испытывают одинаковую разность потенциалов. (См. Рисунок 2, где вольтметр обозначен символом V.)

    Амперметры подключаются последовательно к любому измеряемому устройству. Последовательное соединение используется потому, что последовательно соединенные объекты имеют одинаковый ток, проходящий через них. (См. Рисунок 3, где амперметр обозначен символом A.)

    Рисунок 2. (a) Для измерения разности потенциалов в этой последовательной цепи вольтметр (V) помещают параллельно источнику напряжения или одному из резисторов. Обратите внимание, что напряжение на клеммах измеряется между точками a и b. Невозможно подключить вольтметр напрямую к ЭДС без учета его внутреннего сопротивления, r . (b) Используемый цифровой вольтметр. (предоставлено Messtechniker, Wikimedia Commons) Рис. 3. Амперметр (A) включен последовательно для измерения тока.Весь ток в этой цепи протекает через счетчик. Амперметр будет иметь такие же показания, если он расположен между точками d и e или между точками f и a, как и в показанном положении. (Обратите внимание, что заглавная буква E обозначает ЭДС, а r обозначает внутреннее сопротивление источника разности потенциалов.)

    Аналоговые счетчики имеют стрелку, которая поворачивается, чтобы указывать на числа на шкале, в отличие от цифровых счетчиков , которые имеют числовые показания, подобные портативному калькулятору.Сердцем большинства аналоговых счетчиков является устройство, называемое гальванометром , обозначенное буквой G. Ток, протекающий через гальванометр, [латекс] \ boldsymbol {I _ {\ textbf {G}}} [/ latex], производит пропорциональное отклонение стрелки. . (Это отклонение происходит из-за силы магнитного поля на провод с током.)

    Двумя важнейшими характеристиками данного гальванометра являются его сопротивление и чувствительность по току. Чувствительность по току — это ток, который дает полного отклонения стрелки гальванометра. — максимальный ток, который может измерить прибор.Например, гальванометр с текущей чувствительностью [латекс] \ boldsymbol {50 \; \ mu \ textbf {A}} [/ latex] имеет максимальное отклонение стрелки, когда [латекс] \ boldsymbol {50 \; \ mu \ textbf {A}} [/ latex] проходит через него, считывает половину шкалы, когда [latex] \ boldsymbol {25 \; \ mu \ textbf {A}} [/ latex] проходит через него, и так далее.

    Если такой гальванометр имеет сопротивление [латекс] \ boldsymbol {25 — \; \ Omega} [/ latex], то напряжение только [латекс] \ boldsymbol {V = IR = (50 \; \ mu \ textbf { A}) (25 \; \ Omega) = 1. 25 \; \ textbf {mV}} [/ latex] производит показание полной шкалы. Подключив резисторы к этому гальванометру различными способами, вы можете использовать его как вольтметр или амперметр, который может измерять широкий диапазон напряжений или токов.

    Гальванометр как вольтметр

    На рисунке 4 показано, как гальванометр можно использовать в качестве вольтметра, подключив его последовательно с большим сопротивлением, [латекс] \ boldsymbol {R} [/ латекс]. Значение сопротивления [латекс] \ boldsymbol {R} [/ латекс] определяется максимальным измеряемым напряжением.Предположим, вам нужно 10 В для полного отклонения вольтметра, содержащего [латексный] \ boldsymbol {25 — \; \ Omega} [/ latex] гальванометр с [латексным] \ boldsymbol {50 — \; \ mu \ textbf {A}} [/ latex] чувствительность. Затем 10 В, приложенное к измерителю, должно производить ток [латекс] \ boldsymbol {50 \; \ mu \ textbf {A}} [/ latex]. Общее сопротивление должно быть

    .

    [латекс] \ boldsymbol {R _ {\ textbf {tot}} = R + r =} [/ latex] [latex] \ boldsymbol {\ frac {V} {I}} [/ latex] [latex] \ boldsymbol { =} [/ latex] [латекс] \ boldsymbol {\ frac {10 \; \ textbf {V}} {50 \; \ mu \ textbf {A}}} [/ latex] [латекс] \ boldsymbol {= 200 \ ; \ textbf {k} \ Omega \; \ textbf {или}} [/ latex]

    [латекс] \ boldsymbol {R = R _ {\ textbf {tot}} — r = 200 \; \ textbf {k} \ Omega — 25 \; \ Omega \ приблизительно 200 \; \ textbf {k} \ Omega} [ / латекс]

    ([латекс] \ boldsymbol {R} [/ latex] настолько велик, что сопротивление гальванометра, [латекс] \ boldsymbol {r} [/ latex], почти ничтожно. ) Обратите внимание, что 5 В, приложенное к этому вольтметру, вызывает отклонение в половину шкалы, создавая ток [латекс] \ boldsymbol {25 — \; \ mu \ textbf {A}} [/ latex] через измеритель, и поэтому показания вольтметра пропорционально напряжению по желанию.

    Этот вольтметр не годится для напряжений менее примерно половины вольта, потому что отклонение измерителя будет небольшим и его трудно будет точно прочитать. Для других диапазонов напряжения другие сопротивления устанавливаются последовательно с гальванометром. У многих метров есть выбор шкалы.Этот выбор включает последовательное включение соответствующего сопротивления с гальванометром.

    Рисунок 4. Большое сопротивление R , включенное последовательно с гальванометром G, дает вольтметр, отклонение которого на полную шкалу зависит от выбора R . Чем больше измеряемое напряжение, тем больше должно быть R . (Обратите внимание, что r представляет внутреннее сопротивление гальванометра. )

    Гальванометр как амперметр

    Тот же гальванометр можно превратить в амперметр, разместив его параллельно небольшому сопротивлению [латекс] \ boldsymbol {R} [/ latex], которое часто называют шунтирующим сопротивлением , как показано на рисунке 5. Поскольку шунт сопротивление невелико, большая часть тока проходит через него, что позволяет амперметру измерять токи, намного превышающие те, которые вызывают полное отклонение гальванометра.

    Предположим, например, что необходим амперметр, который дает полное отклонение на 1.0 A, и содержит такой же гальванометр [latex] \ boldsymbol {25 — \; \ Omega} [/ latex] с его чувствительностью [latex] \ boldsymbol {50 — \; \ mu \ textbf {A}} [/ latex] . Поскольку [latex] \ boldsymbol {R} [/ latex] и [latex] \ boldsymbol {r} [/ latex] параллельны, напряжение на них одинаковое.

    Эти [латекс] \ boldsymbol {IR} [/ latex] капли — это [latex] \ boldsymbol {IR = I_Gr} [/ latex], так что [latex] \ boldsymbol {IR = \ frac {I_G} {I} = \ frac {R} {r}} [/ latex]. Решая для [latex] \ boldsymbol {R} [/ latex] и отмечая, что [latex] \ boldsymbol {I_G} [/ latex] — это [latex] \ boldsymbol {50 \; \ mu \ textbf {A}} [/ latex] и [latex] \ boldsymbol {I} [/ latex] равно 0.{-3} \; \ Omega}. [/ Латекс]

    Рисунок 5. Небольшое шунтирующее сопротивление R , размещенное параллельно гальванометру G, дает амперметр, полное отклонение которого зависит от выбора R . Чем больше измеряемый ток, тем меньше должно быть R . Большая часть тока ( I ), протекающего через счетчик, шунтируется через R для защиты гальванометра.(Обратите внимание, что r представляет внутреннее сопротивление гальванометра.) Амперметры также могут иметь несколько шкал для большей гибкости в применении. Различные масштабы достигаются путем переключения различных шунтирующих сопротивлений параллельно гальванометру — чем больше максимальный измеряемый ток, тем меньше должно быть шунтирующее сопротивление.

    Когда вы используете вольтметр или амперметр, вы подключаете другой резистор к существующей цепи и, таким образом, изменяете схему.В идеале вольтметры и амперметры не оказывают заметного влияния на схему, но полезно изучить обстоятельства, при которых они влияют или не влияют.

    Сначала рассмотрим вольтметр, который всегда размещается параллельно с измеряемым устройством. Через вольтметр протекает очень небольшой ток, если его сопротивление на несколько порядков больше, чем сопротивление устройства, и поэтому на цепь это не оказывает заметного влияния. (См. Рисунок 6 (a).) (Большое сопротивление, параллельное малому, имеет суммарное сопротивление, по существу равное малому.) Если, однако, сопротивление вольтметра сопоставимо с сопротивлением измеряемого устройства, то два параллельно подключенных устройства имеют меньшее сопротивление, что существенно влияет на цепь. (См. Рисунок 6 (b).) Напряжение на устройстве не такое, как при отключении вольтметра от цепи.

    Рис. 6. (a) Вольтметр, имеющий сопротивление намного больше, чем устройство ( R Voltmeter >> R ), с которым он подключен параллельно, создает параллельное сопротивление, по существу такое же, как и у устройства, и не оказывает заметного влияния измеряемая цепь.(b) Здесь вольтметр имеет такое же сопротивление, что и устройство ( R Voltmeter ≅ R ), так что параллельное сопротивление составляет половину от того, которое есть, когда вольтметр не подключен. Это пример значительного изменения схемы, которого следует избегать.

    Амперметр подключается последовательно к ветви измеряемой цепи, так что его сопротивление добавляется к этой ветви. Обычно сопротивление амперметра очень мало по сравнению с сопротивлениями устройств в цепи, поэтому дополнительное сопротивление незначительно.(См. Рисунок 7 (a).) Однако, если задействованы очень малые сопротивления нагрузки или если сопротивление амперметра не такое низкое, как должно быть, то общее последовательное сопротивление значительно больше, а ток в ветви измеряется уменьшается. (См. Рисунок 7 (b).)

    Практическая проблема может возникнуть, если амперметр подключен неправильно. Если его подключить параллельно с резистором для измерения тока в нем, вы можете повредить счетчик; низкое сопротивление амперметра позволит большей части тока в цепи проходить через гальванометр, и этот ток будет больше, поскольку эффективное сопротивление меньше.

    Рис. 7. (a) Амперметр обычно имеет такое маленькое сопротивление, что общее последовательное сопротивление в измеряемой ветви существенно не увеличивается. Схема практически не изменилась по сравнению с отсутствием амперметра. (b) Здесь сопротивление амперметра такое же, как сопротивление ветви, так что общее сопротивление удваивается, а ток вдвое меньше, чем без амперметра. Этого существенного изменения схемы следует избегать.

    Одним из решений проблемы вольтметров и амперметров, мешающих измеряемым цепям, является использование гальванометров с большей чувствительностью.Это позволяет создавать вольтметры с большим сопротивлением и амперметры с меньшим сопротивлением, чем при использовании менее чувствительных гальванометров.

    Существуют практические пределы чувствительности гальванометра, но можно получить аналоговые измерители, которые делают измерения с точностью до нескольких процентов. Обратите внимание, что неточность возникает из-за изменения схемы, а не из-за неисправности измерителя.

    Подключения: границы знаний

    Выполнение измерения изменяет измеряемую систему таким образом, что приводит к погрешности измерения.Для макроскопических систем, таких как схемы, обсуждаемые в этом модуле, изменение обычно можно сделать пренебрежимо малым, но полностью исключить его нельзя. Для субмикроскопических систем, таких как атомы, ядра и более мелкие частицы, измерение изменяет систему таким образом, что невозможно сделать сколь угодно малым. Это фактически ограничивает знание системы — даже ограничивает то, что природа может знать о самой себе. Мы увидим глубокие последствия этого, когда принцип неопределенности Гейзенберга будет обсуждаться в модулях по квантовой механике.6} [/ латекс].

    Проверьте свое понимание

    1: Цифровые измерители способны обнаруживать меньшие токи, чем аналоговые измерители, использующие гальванометры. Как это объясняет их способность измерять напряжение и ток более точно, чем аналоговые измерители?

    Исследования PhET: комплект для конструирования цепей (только для постоянного тока), виртуальная лаборатория

    Стимулируйте нейрон и следите за тем, что происходит. Сделайте паузу, перемотайте назад и двигайтесь вперед во времени, чтобы наблюдать за перемещением ионов через мембрану нейрона.

    Рис. 8. Комплект для конструирования цепей (только для постоянного тока), виртуальная лаборатория
    • Вольтметры измеряют напряжение, а амперметры измеряют ток.
    • Вольтметр помещается параллельно источнику напряжения для получения полного напряжения и должен иметь большое сопротивление, чтобы ограничить его влияние на цепь.
    • Амперметр подключается последовательно, чтобы получить полный ток, протекающий через ответвление, и должен иметь небольшое сопротивление, чтобы ограничить его влияние на цепь.
    • Оба могут быть основаны на комбинации резистора и гальванометра, устройства, которое дает аналоговые показания тока.
    • Стандартные вольтметры и амперметры изменяют измеряемую цепь и, таким образом, имеют ограниченную точность.

    Концептуальные вопросы

    1: Почему не следует подключать амперметр непосредственно к источнику напряжения, как показано на рисунке 9? (Обратите внимание, что скрипт E на рисунке означает ЭДС.)

    Рис. 9.

    2: Предположим, вы используете мультиметр (предназначенный для измерения диапазона напряжений, токов и сопротивлений) для измерения тока в цепи и случайно оставляете его в режиме вольтметра.Как измеритель повлияет на схему? Что бы произошло, если бы вы измеряли напряжение, но случайно перевели измеритель в режим амперметра?

    3: Укажите точки, к которым можно подключить вольтметр для измерения следующих разностей потенциалов на Рисунке 10: (a) разность потенциалов источника напряжения; (b) разность потенциалов на [латексе] \ boldsymbol {R_1} [/ latex]; (c) через [латекс] \ boldsymbol {R_2} [/ latex]; (г) поперек [латекса] \ boldsymbol {R_3} [/ latex]; (e) через [латекс] \ boldsymbol {R_2} [/ latex] и [латекс] \ boldsymbol {R_3} [/ latex]. Обратите внимание, что на каждую часть может быть несколько ответов.

    Рисунок 10.

    4: Для измерения токов на рисунке 10 вы замените провод между двумя точками на амперметр. Укажите точки, между которыми вы разместите амперметр, чтобы измерить следующее: (a) общий ток; (б) ток, протекающий через [латекс] \ boldsymbol {R_1} [/ latex]; (c) через [латекс] \ boldsymbol {R_2} [/ латекс]; (г) через [латекс] \ boldsymbol {R_3} [/ латекс]. Обратите внимание, что на каждую часть может быть несколько ответов.

    Проблемные упражнения

    1: Какова чувствительность гальванометра (то есть, какой ток дает полное отклонение) внутри вольтметра, имеющего [латексный] \ boldsymbol {1.00 — \; \ textbf {M} \ Omega} [ / латекс] по шкале 30,0 В?

    2: Какова чувствительность гальванометра (то есть, какой ток дает полное отклонение) внутри вольтметра, имеющего [латексный] \ boldsymbol {25.0 — \; \ textbf {k} \ Omega} [ / латекс] по шкале 100 В?

    3: Найдите сопротивление, которое необходимо подключить последовательно с символом [латекс] \ bold {25.0 — \; \ Omega} [/ latex] гальванометр с чувствительностью [latex] \ boldsymbol {50.0 — \; \ mu \ textbf {A}} [/ latex] (такой же, как тот, который обсуждается в тексте), чтобы позволить его следует использовать как вольтметр с показаниями полной шкалы 0,100 В.

    4: Найдите сопротивление, которое необходимо подключить последовательно с [латексным] \ boldsymbol {25.0 — \; \ Omega} [/ latex] гальванометром, имеющим [латексный] \ boldsymbol {50.0 — \; \ mu \ textbf {A}} [/ latex] чувствительность (такая же, как та, что обсуждается в тексте), позволяющая использовать его в качестве вольтметра с показаниями полной шкалы 3000 В.Включите принципиальную схему в свое решение.

    5: Найдите сопротивление, которое необходимо разместить параллельно [латексному] \ boldsymbol {25.0 — \; \ Omega} [/ latex] гальванометру с [латексным] \ boldsymbol {50.0 — \; \ textbf {A }} [/ latex] чувствительность (такая же, как та, что обсуждается в тексте), позволяющая использовать его в качестве амперметра с показаниями полной шкалы 10,0 A. Включите принципиальную схему в свое решение.

    6: Найдите сопротивление, которое необходимо разместить параллельно символу [латекса] \ bold {25.0 — \; \ Omega} [/ latex] гальванометр с чувствительностью [latex] \ boldsymbol {50.0 — \; \ mu \ textbf {A}} [/ latex] (такой же, как тот, который обсуждается в тексте), чтобы позволить его следует использовать как амперметр с показаниями полной шкалы 300 мА.

    7: Найдите сопротивление, которое необходимо подключить последовательно с [латексным] \ boldsymbol {10.0 — \; \ Omega} [/ latex] гальванометром с [латексным] \ boldsymbol {100 — \; \ mu \ textbf {A}} [/ latex] чувствительность, позволяющая использовать его в качестве вольтметра при: (а) полномасштабном показании 300 В и (б) 0.Полномасштабное показание 300 В.

    8: Найдите сопротивление, которое необходимо разместить параллельно [латексному] \ boldsymbol {10.0 — \; \ Omega} [/ latex] гальванометру с [латексным] \ boldsymbol {100 — \; \ mu \ textbf {A}} [/ latex] чувствительность, позволяющая использовать его в качестве амперметра с: (a) показанием полной шкалы 20,0 A и b) показанием полной шкалы 100 мА.

    9: Предположим, вы измеряете напряжение на клеммах щелочного элемента на 1,585 В, имеющего внутреннее сопротивление [латекс] \ boldsymbol {0.100 \; \ Omega} [/ latex], поместив вольтметр [latex] \ boldsymbol {1.00 — \; \ textbf {k} \ Omega} [/ latex] на его клеммы. (См. Рис. 11.) (а) Какой ток течет? (b) Найдите напряжение на клеммах. (c) Чтобы увидеть, насколько близко измеренное напряжение на клеммах к ЭДС, рассчитайте их отношение.

    Рисунок 11.

    10: Предположим, вы измеряете напряжение на клеммах литиевого элемента на 3.200 В, имеющего внутреннее сопротивление [латекс] \ boldsymbol {5.00 \; \ Omega} [/ латекс], помещая [латекс] \ boldsymbol {1.{-5} \; \ Omega} [/ latex] по шкале 3,00 A и содержит гальванометр [латекс] \ boldsymbol {10.0 — \; \ Omega} [/ latex]. Какая чувствительность у гальванометра?

    12: Вольтметр [латекс] \ boldsymbol {1.00 — \; \ textbf {M} \ Omega} [/ latex] устанавливается параллельно [латексному] \ boldsymbol {75.0 — \; \ textbf {k} \ Omega} [/ latex] резистор в цепи. (а) Нарисуйте принципиальную схему подключения. б) Каково сопротивление комбинации? (c) Если напряжение на комбинации остается таким же, как на [латексе] \ boldsymbol {75.0 — \; \ textbf {k} \ Omega} [/ latex] только резистор, каков процент увеличения тока? (d) Если ток через комбинацию остается таким же, как через резистор [latex] \ boldsymbol {75.0 — \; \ textbf {k} \ Omega} [/ latex], каково процентное снижение напряжения ? (e) Являются ли изменения, обнаруженные в частях (c) и (d), значительными? Обсуждать.

    13: Амперметр [latex] \ boldsymbol {0,0200 — \; \ Omega} [/ latex] последовательно с резистором [latex] \ boldsymbol {10.00 — \; \ Omega} [/ latex] в цепи схема.(а) Нарисуйте принципиальную схему подключения. (b) Рассчитайте сопротивление комбинации. (c) Если напряжение в комбинации остается таким же, каким оно было через резистор [latex] \ boldsymbol {10.00 — \; \ Omega} [/ latex], каков процент уменьшения тока? (d) Если ток остается таким же, как через резистор [latex] \ boldsymbol {10.00 — \; \ Omega} [/ latex], то каков процент увеличения напряжения? (e) Являются ли изменения, обнаруженные в частях (c) и (d), значительными? Обсуждать.

    14: Необоснованные результаты

    Предположим, у вас есть гальванометр [latex] \ boldsymbol {40.0 — \; \ Omega} [/ latex] с чувствительностью [latex] \ boldsymbol {25.0 — \; \ mu \ textbf {A}} [/ latex]. (a) Какое сопротивление вы бы включили последовательно, чтобы его можно было использовать в качестве вольтметра с полным отклонением на 0,500 мВ? б) Что неразумного в этом результате? (c) Какие допущения ответственны?

    15: Необоснованные результаты

    (a) Какое сопротивление вы бы поставили параллельно с символом [латекс] \ bold {40.0 — \; \ Omega} [/ latex] гальванометр с чувствительностью
    [латекс] \ boldsymbol {25.0 — \; \ mu \ textbf {A}} [/ latex], позволяющий использовать его в качестве амперметра с полное отклонение для [латекса] \ boldsymbol {10.0 — \; \ mu \ textbf {A}} [/ latex]? б) Что неразумного в этом результате? (c) Какие допущения ответственны?

    Глоссарий

    вольтметр
    прибор для измерения напряжения
    амперметр
    прибор для измерения силы тока
    аналоговый счетчик
    Измерительный прибор, дающий показания в виде движения стрелки над отмеченным датчиком
    цифровой счетчик
    Измерительный прибор, дающий показания в цифровом виде
    гальванометр
    аналоговое измерительное устройство, обозначенное буквой G, которое измеряет ток, используя отклонение стрелки, вызванное силой магнитного поля, действующей на токопроводящий провод
    чувствительность по току
    максимальный ток, который может считывать гальванометр
    полный прогиб
    максимальное отклонение стрелки гальванометра, также известное как чувствительность по току; гальванометр с полным отклонением [латекс] \ boldsymbol {50 \; \ mu \ textbf {A}} [/ latex] имеет максимальное отклонение стрелки, когда [латекс] \ boldsymbol {50 \; \ mu \ textbf {A}} [/ latex] проходит через него
    шунтирующее сопротивление
    небольшое сопротивление [латекс] \ boldsymbol {R} [/ latex], помещенное параллельно гальванометру G для получения амперметра; чем больше измеряемый ток, тем меньше должен быть [латекс] \ boldsymbol {R} [/ latex]; большая часть тока, протекающего через счетчик, шунтируется через [латекс] \ boldsymbol {R} [/ latex] для защиты гальванометра

    Решения

    Проверьте свое понимание

    1: Поскольку цифровые счетчики требуют меньшего тока, чем аналоговые, они изменяют схему меньше, чем аналоговые счетчики.{-4} \; \ Omega} [/ латекс]

    7: (a) [латекс] \ boldsymbol {3.00 \; \ textbf {M} \ Omega} [/ latex]

    (b) [латекс] \ boldsymbol {2.99 \; \ textbf {k} \ Omega} [/ latex]

    9: (a) 1,58 мА
    (b) 1,5848 В (необходимо четыре цифры, чтобы увидеть разницу)

    (c) 0,99990 (нужно пять цифр, чтобы увидеть разницу от единицы)

    11: [латекс] \ boldsymbol {15.0 \; \ mu \ textbf {A}} [/ латекс]

    13: (а)

    Рисунок 12.{-1}} [/ latex] процентное увеличение

    (e) Не имеет значения.

    15: (a) [латекс] \ boldsymbol {-66.7 \; \ Omega} [/ латекс]

    (b) У вас не может быть отрицательного сопротивления.

    (c) Неразумно, что [latex] \ boldsymbol {I_G} [/ latex] больше, чем [latex] \ boldsymbol {I _ {\ textbf {tot}}} [/ latex] (см. Рисунок 5). Вы не можете добиться полного отклонения, используя ток, меньший, чем чувствительность гальванометра.

    Вольтметры и амперметры

    Вольтметры и амперметры измеряют соответственно напряжение и ток в цепи.Некоторые измерители в автомобильных приборных панелях, цифровых камерах, сотовых телефонах и тюнерах-усилителях являются вольтметрами или амперметрами.

    Вольтметры

    Вольтметр — это прибор, который измеряет разность электрических потенциалов между двумя точками в электрической цепи. Аналоговый вольтметр перемещает указатель по шкале пропорционально напряжению в цепи; цифровой вольтметр обеспечивает числовой дисплей. Любое измерение, которое можно преобразовать в напряжение, можно отобразить на правильно откалиброванном измерителе; такие измерения включают давление, температуру и расход.

    Вольтметр

    Вольтметр демонстрационный из класса физики

    Чтобы вольтметр мог измерять напряжение устройства, он должен быть подключен параллельно этому устройству. Это необходимо, потому что параллельные объекты испытывают одинаковую разность потенциалов.

    Вольтметр параллельно

    (a) Для измерения разности потенциалов в этой последовательной цепи вольтметр (V) помещают параллельно источнику напряжения или одному из резисторов.Обратите внимание, что напряжение на клеммах измеряется между точками a и b. Невозможно подключить вольтметр напрямую к ЭДС без учета его внутреннего сопротивления r. (b) Используемый цифровой вольтметр

    Амперметры

    Амперметр измеряет электрический ток в цепи. Название происходит от названия единицы измерения электрического тока в системе СИ, ампер (А).

    Чтобы амперметр мог измерять ток устройства, он должен быть последовательно подключен к этому устройству.Это необходимо, поскольку последовательно соединенные объекты испытывают одинаковый ток. Их нельзя подключать к источнику напряжения — амперметры предназначены для работы с минимальной нагрузкой (которая относится к падению напряжения на амперметре, обычно составляющему небольшую долю вольта).

    Амперметр серии

    Амперметр (A) включен последовательно для измерения тока. Весь ток в этой цепи протекает через счетчик. Амперметр будет иметь такие же показания, если он расположен между точками d и e или между точками f и a, как и в показанном положении.(Обратите внимание, что заглавная буква E обозначает ЭДС, а r обозначает внутреннее сопротивление источника разности потенциалов.)

    Гальванометры (аналоговые счетчики)

    У аналоговых счетчиков есть стрелки, которые поворачиваются, чтобы указывать на числа на шкале, в отличие от цифровых счетчиков, которые имеют числовые показания. Сердцем большинства аналоговых счетчиков является устройство, называемое гальванометром, обозначаемое G . Ток через гальванометр I G вызывает пропорциональное движение или отклонение стрелки.

    Двумя важнейшими характеристиками любого гальванометра являются его сопротивление и чувствительность по току. Чувствительность по току — это ток, который дает полное отклонение стрелки гальванометра, другими словами, максимальный ток, который может измерить прибор. Например, гальванометр с токовой чувствительностью 50 мкА имеет максимальное отклонение стрелки при протекании через него 50 мкА, находится на полпути шкалы, когда через него протекает 25 мкА, и так далее.

    Если такой гальванометр имеет сопротивление 25 Ом, то напряжение всего В = IR = (50 мкА) (25 Ом) = 1.25 мВ дает показания полной шкалы. Подключив резисторы к этому гальванометру различными способами, вы можете использовать его как вольтметр или амперметр для измерения широкого диапазона напряжений или токов.

    Гальванометры как вольтметры

    Гальванометр может работать как вольтметр, если он подключен последовательно с большим сопротивлением R . Значение R определяется максимальным измеряемым напряжением. Предположим, вам нужно 10 В для полного отклонения вольтметра, содержащего гальванометр с сопротивлением 25 Ом и чувствительностью 50 мкА.Тогда приложенное к измерителю напряжение 10 В должно давать ток 50 мкА. Общее сопротивление должно быть:

    $ R_ {tot} = R + r = \ frac {V} {I} = \ frac {10V} {50 \ mu A} = 200 k \ Omega,

    $

    или:

    $ R = R_ {tot} — r = 200 k \ Omega — 25 \ Omega \ приблизительно 200 k \ Omega. $

    (R настолько велик, что сопротивление гальванометра, r, почти пренебрежимо мало). Обратите внимание, что приложено 5 В. Этот вольтметр производит отклонение на половину шкалы, пропуская через измеритель ток 25 мкА, и поэтому показания вольтметра пропорциональны напряжению, если это необходимо.Этот вольтметр не будет полезен для напряжений менее примерно половины вольта, потому что отклонение измерителя будет слишком маленьким для точного считывания. Для других диапазонов напряжения другие сопротивления устанавливаются последовательно с гальванометром. Многие измерители позволяют выбирать шкалы, которые включают последовательное включение соответствующего сопротивления с гальванометром.

    Гальванометры как амперметры

    Тот же гальванометр может также работать как амперметр, если он размещен параллельно с небольшим сопротивлением R , часто называемым шунтирующим сопротивлением.Поскольку сопротивление шунта невелико, большая часть тока проходит через него, что позволяет амперметру измерять токи, намного превышающие те, которые вызывают полное отклонение гальванометра.

    Предположим, например, что нам нужен амперметр, который дает полную шкалу отклонения для 1,0 А и который содержит тот же гальванометр 25 Ом с чувствительностью 50 мкА. Поскольку R и R параллельны, напряжение на них одинаковое.

    Эти ИК-капли следующие: IR = I G r

    , так что: $ IR = \ frac {I_G} {I} = \ frac {R} {r}.{-3} \ Omega. $

    Разница между вольтметром и мультиметром

    Разность потенциалов, ток и сопротивление — это три основных параметра, которые связаны с любой электронной или электрической цепью. Все три параметра можно количественно измерить с помощью подходящих инструментов. Вольтметр — это электрический прибор, используемый для измерения разности потенциалов между двумя точками в электрической или электронной цепи. Ток можно измерить амперметром, а сопротивление можно рассчитать математически на основе известных значений напряжения и тока или измерить напрямую с помощью диода или мультиметра.

    Фактически, мультиметр может напрямую измерять все три вышеупомянутых свойства. Следовательно, мультиметр — это один из электрических инструментов, который может напрямую измерять разность потенциалов, ток, а также сопротивление электрической цепи. Некоторые мультиметры могут также измерять несколько других параметров: частоту канала, емкость, индуктивность, влажность и т. Д. Сходства и различия между вольтметром и мультиметром обсуждаются в следующих разделах.

    • И вольтметр, и мультиметр являются электрическими приборами и поэтому могут работать только при наличии электричества в цепи.
    • Оба прибора могут измерять разность потенциалов между двумя точками электрической цепи в вольтах или милливольтах.
    • Оба они имеют аналоговую и цифровую версии.
    Мультиметры
    Вольтметр Мультиметр
    Вольтметр может измерять только разность потенциалов между двумя точками в электрической цепи. Мультиметр — это многоцелевой прибор, который может напрямую измерять различные параметры, такие как разность потенциалов, ток, сопротивление, частоту, емкость и т. Д.
    Для измерения сопротивления цепи вместе с вольтметром требуется амперметр. Мультиметр может напрямую измерять сопротивление цепи. Так что нет необходимости использовать для этого отдельный амперметр.
    Обычно не используется для проверки диодов или транзисторов. Мультиметр можно использовать для проверки диодов и транзисторов.
    Вольтметр не может заменить мультиметр. Мультиметр можно заменить вольтметром.
    Вольтметры сравнительно дешевле. Мультиметры более дорогие, поскольку состоят из нескольких частей для измерения различных параметров.
    Область применения вольтметра узкая. имеют широкую область применения благодаря своей способности выполнять самые разные функции.
    Обзор электрических схем

    — Ответы № 3

    Обзор электрических цепей


    Переход к:

    Главная страница сеанса обзора — Список тем

    Electric Circuits — Главная || Версия для печати || Вопросы со ссылками

    Ответы на вопросы: Все || # 1-7 || # 8-51 || # 52-59 || # 60-72

    Часть C: Диаграммы и анализ

    52.Рассмотрите диаграмму справа, чтобы ответить на следующие вопросы.

    а. Если 4 кулоны заряда проходят мимо точки A за 2 секунды, то ___ кулонов заряда проходят мимо точки B за 2 секунды.

    а. менее 4

    г. 4

    г. более 4

    г. Невозможно сделать такой прогноз без знания сопротивлений.

    г. Если 4 кулоны заряда проходят мимо точки A за 2 секунды, то ___ кулонов заряда проходят мимо точки B за 1 секунду.

    а. менее 4

    г. 4

    г. более 4

    г. Невозможно сделать такой прогноз без знания сопротивлений.

    г. Если 4 кулоны заряда проходят мимо точки A за 2 секунды, то ___ кулонов заряда проходят мимо точки B за 4 секунды.

    а. менее 4

    г. 4

    г. более 4

    г. Невозможно сделать такой прогноз без знания сопротивлений.

    г. Если 4 кулоны заряда проходят мимо точки A за 2 секунды, то ___ кулонов заряда проходят мимо точки B за 4 секунды.

    e. Если 4 кулоны заряда проходят мимо точки A за 2 секунды, то ___ кулонов заряда проходят мимо точки C за 4 секунды.

    ф. Предположим, что сопротивление лампочки, расположенной между точками A и B, увеличивается. Это приведет к тому, что ток через другую лампочку станет ____ (увеличение, уменьшение, останется прежним).

    Ответ: См. Ответы ниже.

    Этот вопрос проверяет ваше понимание тока как скорости, с которой заряд (выраженный здесь в кулонах) проходит через точку в цепи. Ток определяется как отношение заряда / времени. В такой последовательной цепи, как эта, ток везде одинаковый.

    а. B ; ток 2,0 ампера в точке А. То же самое 2.0 ампер в точке B, 4 кулоны должны пройти точку за 2 секунды.

    г. А ; ток составляет 2,0 ампера в точке A. Чтобы получить такие же 2,0 ампера в точке B, 2 кулоны должны пройти через точку за 1 секунду. За меньшее количество времени меньше заряда будет проходить мимо точки.

    г. C ; ток составляет 2,0 ампера в точке A. Чтобы получить те же 2,0 ампера в точке B, 8 кулонов должны пройти через точку за 4 секунды. Чем больше времени, тем больше заряда будет проходить мимо точки.

    г. 8 ; ток составляет 2,0 ампера в точке A. Чтобы быть таким же 2,0 ампера в точке B, 8 Кулоны должны пройти точку B за 4 секунды.

    e. 8 ; ток составляет 2,0 ампера в точке A. Чтобы быть такими же 2,0 ампера в точке B, 8 Кулоны должны пройти точку C за 4 секунды.

    ф. уменьшение ; увеличение сопротивления лампочки для ряда последовательных лампочек увеличит общее сопротивление цепи и вызовет уменьшение тока во всей цепи.

    53. На схеме справа показаны три идентичные лампочки, соединенные последовательно. Несколько точек на контуре обозначены буквами. Сравните электрический потенциал и электрическую потенциальную энергию различных точек. Для каждого сравнения используйте символ больше (>), меньше (<) или приблизительно равно (=).

    Электрический потенциал

    Сравнение

    Потенциальная энергия

    Сравнение

    В А = В В PE A = PE B
    V B > V C PE B > PE C
    V C = V D PE C = PE D
    V D > V E PE D > PE E
    V E = V F PE E = PE F
    V F > V G PE F > PE G
    V G = V H PE G = PE H
    В В < В А PE H < PE A

    Ответ: См. Таблицу выше.

    Точка А соответствует положительной клемме аккумулятора. В точке А заряд обладает наибольшим количеством электрической потенциальной энергии. В этом месте высокое напряжение. В процессе прохождения по цепи к клемме — электрическая потенциальная энергия заряда преобразуется в световую энергию (и тепловую энергию) в лампочках. Он будет терять части своей электроэнергии в каждой лампочке. Лампочка просто служит устройством преобразования энергии, которое преобразует электрическую потенциальную энергию в неэлектрическую форму.Таким образом, потенциальная энергия (и, следовательно, напряжение) будет падать в значении каждый раз, когда проходит мимо лампочки и достигает точки, находящейся дальше от клеммы высокой энергии +. Наконец, в точке H вся электрическая энергия и напряжение были потеряны, и для заряда требуется подвести некоторую энергию от батареи.

    54. Используйте соответствующие условные обозначения, чтобы построить схему цепи, питаемой от 6-вольтовой батареи, которая состоит из двух последовательно соединенных резисторов на 3 Ом.Поместите амперметры последовательно в таком месте, чтобы можно было измерить ток через каждый резистор, и в таком месте, чтобы можно было измерить общий ток в цепи. На принципиальной схеме используйте непрерывную стрелку, чтобы указать направление обычного тока. Наконец, укажите на диаграмме показания амперметра.

    Ответ: См. Схему ниже.

    Поскольку два резистора включены последовательно, полное или эквивалентное сопротивление является просто суммой отдельных сопротивлений.Общее сопротивление 6 Ом. Полный ток в цепи можно найти по отношению напряжения батареи к общему сопротивлению:

    I TOT = V TOT / R TOT = (6 В) / (6 Ом) = 1 ампер

    Поскольку это последовательная цепь, ток через батарею такой же, как ток через каждый из резисторов.


    [# 52 | # 53 | # 54 | # 55 | # 56 | # 57 | # 58 | # 59]

    55.Используйте соответствующие условные обозначения, чтобы построить схему цепи, питаемой от 6-вольтовой батареи, которая состоит из двух резисторов на 3 Ом, соединенных параллельно. Поместите амперметр последовательно с каждым из отдельных резисторов таким образом, чтобы можно было измерить ток через каждый резистор. Разместите третий амперметр в таком месте, чтобы можно было измерить общий ток в цепи. На принципиальной схеме используйте непрерывную стрелку, чтобы указать направление обычного тока. Наконец, укажите на диаграмме показания амперметра.

    Ответ: См. Схему ниже.

    Напряжение на каждой ветви эквивалентно напряжению батареи. Ток через ветвь — это просто отношение V / R, где V = 6 Вольт и R = 3 Ом. Этот расчет приводит к выводу, что ток в каждой ветви составляет 2 Ампер. Ток вне ветвей — это просто сумма тока в ветвях. Таким образом, вне ответвлений будет ток 4 Ампер.

    В качестве альтернативы этому решению можно сначала определить эквивалентное сопротивление, используя уравнение 1 / R Tot для параллельных цепей. Эквивалентное сопротивление 1,5 Ом. После определения общий ток цепи можно рассчитать, используя I Tot = V Tot / R Tot . Общий ток цепи составит 4 ампера. Затем разделите 4 А на два равных тока ответвления, чтобы определить ток на каждом из отдельных резисторов.


    [# 52 | # 53 | # 54 | # 55 | # 56 | # 57 | # 58 | # 59]

    56. Рассмотрим схему справа от последовательной цепи. Каждая лампочка в цепи имеет одинаковое сопротивление. Используйте отмеченные точки на схеме, чтобы ответить на следующие вопросы. На каждый вопрос может быть один, менее одного или более одного ответа.

    а. Электрический потенциал в точке A такой же, как электрический потенциал в точке (ах) B .Включите все, что применимо, если применимо.

    г. Электрический потенциал в точке C такой же, как электрический потенциал в точках D и E . Включите все, что применимо, если применимо.

    г. Электрический потенциал в точке F такой же, как электрический потенциал в точке (ах) G . Включите все, что применимо, если применимо.

    г. Электрический потенциал в точке I такой же, как электрический потенциал в точке (ах) H . Включите все, что применимо, если применимо.

    e. Разность электрических потенциалов между точками A и B такая же, как разность электрических потенциалов между точками C, и D (и C и E; и D и E; и F и G; и H и I) . Включите все, что применимо, если применимо.

    ф. Разность электрических потенциалов между точками A и C такая же, как разность электрических потенциалов между точками E и F (и C и F; и D и F; и E и G; и C и G; и D и G; и G и H; и F и H; и G и I; и F и I) .Включите все, что применимо, если применимо.

    г. Разность электрических потенциалов между точками A и F такая же, как и разность электрических потенциалов между точками E, и H (и между любыми другими двумя наборами точек, которые находятся на противоположной стороне двух соседних ламп … например, точки C и H или точки D и H или точки E и I и т. Д.) . Включите все, что применимо, если применимо.

    ч. Разность электрических потенциалов между точками D и H такая же, как и разность электрических потенциалов между точками A и F (и между любыми другими двумя наборами точек, которые находятся на противоположной стороне двух соседних лампочек… например, точки C и H или точки D и I, точки E и I и т. д.) . Включите все, что применимо, если применимо.

    и. Ток в точке A такой же, как ток в точке (ах) C (и в любой другой точке цепи) . Включите все, что применимо, если применимо.

    Дж. Ток в точке E такой же, как ток в точке (ах) F (и в любой другой точке цепи) . Включите все, что применимо, если применимо.

    к. Ток в точке G такой же, как ток в точке (ах) G (и в любой другой точке цепи) .Включите все, что применимо, если применимо.

    Ответ: См. Ответы выше.

    В электрической цепи электрический потенциал движущегося заряда накапливается в батарее и теряется в лампочке (или каком-либо резисторе во внешней цепи). Таким образом, электрический потенциал заряда одинаков для любых двух точек, которые не разделены батареей или лампочкой. (от a до d)

    В этой цепи лампочки имеют одинаковое сопротивление; таким образом, каждая лампочка вызывает одинаковое падение потенциала (разность электрических потенциалов).Таким образом, разность электрических потенциалов будет одинаковой между любыми двумя точками, которые находятся на расстоянии одной или двух лампочек. (с e по h)

    Наконец, в последовательной цепи ток одинаков во всех точках цепи. Поскольку заряд сохраняется и поскольку в цепи нет места, где происходит накопление заряда, во всех местах должна быть одинаковая скорость потока заряда. (от i до k)

    [# 52 | # 53 | # 54 | # 55 | # 56 | # 57 | # 58 | # 59]

    57.Рассмотрим схему справа от параллельной цепи. Каждая лампочка в цепи имеет одинаковое сопротивление. Используйте отмеченные точки на схеме, чтобы ответить на следующие вопросы. На каждый вопрос может быть один, менее одного или более одного ответа.

    а. Электрический потенциал в точке A такой же, как электрический потенциал в точке (точках) B (и в любой другой точке перед лампочками … например, в точках C, D, E и F) . Включите все, что применимо, если применимо.

    г.Электрический потенциал в точке D такой же, как электрический потенциал в точке (точках) B (и в любой другой точке перед лампочками … например, в точках A, C, E и F) . Включите все, что применимо, если применимо.

    г. Электрический потенциал в точке J такой же, как электрический потенциал в точке (точках) K (и в любой другой точке после лампочек … например, в точках G, H, I и L . Включите все, что применимо, если применимо.

    г. Разность электрических потенциалов между точками A и J такая же, как и разность электрических потенциалов между точками B, и K (и любой другой комбинацией двух точек, расположенных на противоположных сторонах лампочек… например, точки A и K, или точки A и L, или точки F и G, или точки D и G, и т. д. и т. д.) . Включите все, что применимо, если применимо.

    e. Разность электрических потенциалов между точками D и G такая же, как и разность электрических потенциалов между точками A и J (и любой другой комбинацией двух точек, расположенных на противоположных сторонах лампочек … например, точек A и K, или точки A и L, или точки F и G, или точки E и H, и т. д. и т. д.) . Включите все, что применимо, если применимо.

    ф. Ток в точке A такой же, как ток в точке (ах) Дж . Включите все, что применимо, если применимо.

    г. Ток в точке B такой же, как ток в точке (ах) K . Включите все, что применимо, если применимо.

    ч. Ток в точке C такой же, как ток в точке (ах) L (и точках F, I, E, H, D и G) . Включите все, что применимо, если применимо.

    и.Ток в точке D такой же, как ток в точке (ах) G (и в точках F, I, C, E и H) . Включите все, что применимо, если применимо.

    Дж. Если бы лампочку, расположенную между точками D и G, заменить лампой с большим сопротивлением, то ток в точках A, D, G и J уменьшился бы. Включите все, что применимо, если применимо.

    к. Если бы лампочку, расположенную между точками D и G, заменить лампочкой с большим сопротивлением, то разность электрических потенциалов между точками и увеличилась бы.Включите все, что применимо, если применимо. (не применимо)

    л. Если бы лампочка, расположенная между точками D и G, до погасла , то ток уменьшился бы в точках A, D, G и J . Включите все, что применимо, если применимо.

    Ответ: См. Ответы выше.

    В электрической цепи электрический потенциал движущегося заряда накапливается в батарее и теряется в лампочке (или каком-либо резисторе во внешней цепи).Таким образом, электрический потенциал заряда одинаков для любых двух точек, которые не разделены батареей или лампочкой. Даже если цепь представляет собой параллельную цепь, любая точка между положительной клеммой аккумулятора и лампочкой будет иметь одинаковый электрический потенциал; и любая точка, расположенная между минусовой клеммой батареи и местом после прохождения через резистор ответвления, имеет одинаковый электрический потенциал. (от a до e)

    В этой цепи лампочки имеют одинаковое сопротивление.Таким образом, когда заряд достигает места разветвления, равное количество заряда выберет среднюю ветвь, как левую ветвь и правую ветвь. Таким образом, при одинаковом сопротивлении ток в каждой ветви одинаков. И ток до места разветвления и после соединения ветвей одинаков. Наконец, точки B и K находятся в местах, где будет проходить ток для двух из трех ветвей; эти точки будут иметь одинаковый ток. (с e по i)

    In j удаление лампочки в первой ветви не повлияет на ток в других ветвях.Такая модификация только снизит общий ток цепи. Меньшее количество ответвлений приведет к большему общему сопротивлению и меньшему общему току. Тем не менее, ток через вторую ветвь по-прежнему представляет собой падение напряжения на второй ветви (которое является напряжением батареи), деленное на сопротивление второй ветви. Поскольку удаление лампочки в первой ветви не меняет ни одной из величин, ток в средней ветви не изменяется.

    В k, электрический потенциал на ветви просто равен напряжению батареи.Удаление лампочки не изменит напряжение батареи. Таким образом, эффекта нет.

    В l, если лампочка в первом ответвлении перегорит, произойдет тот же эффект, что и в части j.


    58. Рассмотрим приведенную ниже схему последовательной цепи. Для каждого резистора используйте стрелки, чтобы указать два места, где нужно было бы отводить провода вольтметра, чтобы измерить падение напряжения на отдельном резисторе.Наконец, укажите показания амперметра и значения напряжения.

    Ответ: См. Диаграмму выше.

    Общее сопротивление (или эквивалентное сопротивление) можно сначала определить с помощью уравнения для последовательных цепей.

    R Tot = R 1 + R 2 + R 3 = 5 Ом + 10 Ом + 15 Ом = 30 Ом

    Как только известно, значение R Tot можно использовать с напряжением батареи (ΔV Tot ) для определения общего тока в цепи.

    I Tot = (ΔV Tot ) / (R Tot ) = (120 В) / (30 Ом) = 4 А

    Для последовательной цепи ток через каждый резистор равен полному току цепи. Таким образом, I 1 = I 2 = I 3 = 4 Ампер.

    Падение напряжения на резисторе можно определить с помощью вольтметра, постукивая выводами металлических проводов на противоположных сторонах резистора. Таким образом, вольтметр определяет разность напряжений (т.е.например, падение напряжения или разность электрических потенциалов) между двумя точками, в которых проводились ответвления. В этой схеме ожидаемые падения напряжения (ΔV 1 , ΔV 2 и ΔV 3 ) можно вычислить, определив произведение IR для каждого резистора. Это показано ниже.

    ΔV 1 = I 1 • R 1 = (4 А) • (5 Ом) = 20 Вольт

    ΔV 2 = I 2 • R 2 = (4 А) • (10 Ом) = 40 Вольт

    ΔV 3 = I 3 • R 3 = (4 А) • (15 Ом) = 60 Вольт


    59.Рассмотрим приведенную ниже схему параллельной цепи. Для каждого резистора используйте стрелки, чтобы указать два места, где нужно было бы отводить провода вольтметра, чтобы измерить падение напряжения на отдельном резисторе. Наконец, укажите показания амперметра и значения напряжения.

    Ответ: См. Диаграмму выше.

    Общее сопротивление (или эквивалентное сопротивление) можно сначала определить с помощью уравнения для последовательных цепей.

    1 / R Tot = 1 / R 1 + 1 / R 2 + 1 / R 3 = 1 / (5 Ом + 1 / (10 Ом) + 1 / (15 Ом)

    R Tot = 2,727 Ом

    Как только известно, значение R Tot можно использовать с напряжением батареи (ΔV Tot ) для определения общего тока в цепи.

    I Tot = (ΔV Tot ) / (R Tot ) = (120 В) / (2,727 Ом) = 44.0 ампер

    Падение напряжения на резисторе можно определить с помощью вольтметра, постукивая выводами металлических проводов на противоположных сторонах резистора. Таким образом, вольтметр определяет разницу в напряжении (то есть падение напряжения или разность электрических потенциалов) между двумя точками, в которых проводились ответвления. Для последовательной цепи ожидаемое падение напряжения на каждом резисторе (ΔV 1 , ΔV 2 и ΔV 3 ) совпадает с общим падением напряжения.Таким образом, ΔV 1 = ΔV 2 = ΔV 3 = 120 Вольт.

    В этой схеме токи ответвления можно вычислить, используя уравнение ΔV = I • R для каждого резистора. Это показано ниже.

    R 1 = ΔV 1 / R 1 = (120 Вольт) / (5 Ом) = 24 Ампер

    I 2 = ΔV 2 / R 2 = (120 Вольт) / (10 Ом) = 12 Ампер

    I 3 = ΔV 3 / R 3 = (120 Вольт) / (15 Ом) = 8 Ампер


    Переход к:

    Главная страница сеанса обзора — Список тем

    Electric Circuits — Главная || Версия для печати || Вопросы со ссылками

    Ответы на вопросы: Все || # 1-7 || # 8-51 || # 52-59 || # 60-72

    Вам тоже может понравиться…

    Пользователи The Review Session часто ищут учебные ресурсы, которые предоставляют им возможности для практики и обзора, которые включают встроенную обратную связь и инструкции. Если это то, что вы ищете, то вам также может понравиться следующее:
    1. Блокнот калькулятора

      Блокнот калькулятора включает в себя текстовые задачи по физике, организованные по темам. Каждая проблема сопровождается всплывающим ответом и аудиофайлом, в котором подробно объясняется, как подойти к проблеме и решить ее.Это идеальный ресурс для тех, кто хочет улучшить свои навыки решения проблем.

      Посещение: Панель калькулятора На главную | Блокнот для калькулятора — электрические схемы

    2. Minds On Physics App Series

      Minds On Physics the App («MOP the App») представляет собой серию интерактивных модулей вопросов для учащихся, которые серьезно настроены улучшить свое концептуальное понимание физики. Каждый модуль этой серии посвящен отдельной теме и разбит на подтемы.«Опыт MOP» предоставит учащемуся сложные вопросы, отзывы и помощь по конкретным вопросам в контексте игровой среды. Он доступен для телефонов, планшетов, Chromebook и компьютеров Macintosh. Это идеальный ресурс для тех, кто желает усовершенствовать свои способности к концептуальному мышлению. Четвертая часть серии включает темы «Электрические схемы».

      Посетите: MOP the App Home || MOP приложение — часть 4

    Физика — Электродвижущая сила — Бирмингемский университет

    Электродвижущая сила (ЭДС) равна разности потенциалов на клеммах при отсутствии тока.ЭДС и разность потенциалов на клеммах ( В, ) измеряются в вольтах, но это не одно и то же. ЭДС ( ϵ ) — это количество энергии ( E ), обеспечиваемое батареей на каждый проходящий кулон заряда ( Q ).

    Как рассчитать ЭДС?

    ЭДС можно записать через внутреннее сопротивление батареи ( r ) где: ϵ = I (r + R )

    Что из закона Ома, мы можем затем изменить это в терминах оконечного сопротивления: ϵ = В + Ir

    ЭДС ячейки может быть определена путем измерения напряжения на ячейке с помощью вольтметра и тока в цепи с помощью амперметра для различных сопротивлений.Затем мы можем настроить схему для определения ЭДС, как показано ниже.

    ЭДС и внутреннее сопротивление электрических элементов и батарей

    Исследование ЭМП

    Как закон Фарадея соотносится с ЭМП?

    Закон Фарадея гласит, что любое изменение магнитного поля катушки будет индуцировать в катушке ЭДС (а следовательно, и ток). Он пропорционален минус скорости изменения магнитного потока ( ϕ ) (примечание: N — количество витков в катушке).

    Используя закон Фарадея, общество извлекло выгоду из таких важных технологий, как трансформаторы, которые используются для передачи электроэнергии в национальной энергосистеме Великобритании, которая теперь является необходимостью в наших домах. Также он используется в электрических генераторах и двигателях, таких как плотины гидроэлектростанций, которые производят электричество, которое сейчас является неотъемлемой частью наших современных технологических потребностей. Текущий исследовательский проект MAG-DRIVE в Бирмингеме направлен на поиск способов разработки и улучшения материалов с постоянными магнитами, которые могут быть использованы в электромобилях следующего поколения.ЭМП также генерируется солнечными батареями, поэтому они важны для исследований в области возобновляемых источников энергии.

    Лаборатория признаний

    Исследователи подкаста In the Laboratory Confessions рассказывают о своем лабораторном опыте в контексте практических экзаменов A Level. Эпизоды, посвященные правильному использованию цифровых инструментов (простое гармоническое движение), правильному построению принципиальных схем (удельное сопротивление в проводе) и использованию источников питания постоянного тока (конденсаторов), имеют отношение к эксперименту по ЭДС, ниже вы можете услышать удельное сопротивление. в проводном подкасте.

    Как мы интерпретируем наши данные?

    По мере увеличения сопротивления переменного резистора величина тока будет уменьшаться. График зависимости напряжения от тока должен давать линейную зависимость, где градиент линии дает отрицательное внутреннее сопротивление ячейки ( -r ), а точка пересечения дает ЭДС (напряжение, при котором ток равен 0).

    Выполнение нескольких измерений при разных значениях сопротивления даст больше точек на графике V-I, что сделает подбор более надежным.Также рекомендуется повторить измерения, так как ячейка будет постепенно стекать, что повлияет на показания. Во избежание разряда элемента / батареи ее следует отключать между измерениями. В качестве альтернативы в схему можно включить выключатель. Также не рекомендуется использовать аккумуляторные батареи, так как они имеют низкое внутреннее сопротивление.

    Хотя этот эксперимент довольно прост, он поможет вам отличить конечную разницу от ЭДС, что может быть трудной для понимания концепцией.Поскольку люди становятся все более зависимыми от электричества, исследования, связанные с ЭМП, важны для развития и технического прогресса электричества.

    Следующие шаги

    Эти ссылки предоставлены только для удобства и в информационных целях; они не означают одобрения или одобрения Бирмингемским университетом какой-либо информации, содержащейся на внешнем веб-сайте. Бирмингемский университет не несет ответственности за точность, законность или содержание внешнего сайта или последующих ссылок.Пожалуйста, свяжитесь с внешним сайтом для получения ответов на вопросы относительно его содержания.

    Как вольтметр подключен к цепи для измерения физики класса 12 CBSE

    Совет : Вольтметр — это устройство, которое измеряет разность потенциалов в двух точках в цепи. Подумайте, если мы хотим измерить разность потенциалов на компоненте или в любых двух точках цепи, то следует ли подключать вольтметр последовательно и параллельно через эти две точки.

    Полный пошаговый ответ:
    Вольтметр — это устройство, которое измеряет разность потенциалов в двух точках цепи. Он подключается параллельно к двум точкам цепи. Он должен быть подключен параллельно, а не последовательно, потому что мы хотим измерить разность потенциалов между двумя точками разности. Если мы подключим последовательно вольтметр, он будет измерять разность потенциалов в 0 вольт. Другими словами замыкаем клемму вольтметра.


    Дополнительная информация: Разберемся с механизмом или настройкой вольтметра.
    Вольтметр состоит из гальванометра. Гальванометр — это устройство, определяющее ток. Однако гальванометр имеет меньшую устойчивость к току. Максимальный ток, который может пропускать гальванометр, обозначается как $ {{i} _ {g}} $.
    К гальванометру последовательно подключено большое сопротивление.

    Это потому, что ток обратно пропорционален сопротивлению.Поскольку гальванометр может принимать небольшой ток, большое последовательное сопротивление увеличивает общее сопротивление устройства, и, таким образом, очень небольшое количество тока будет проходить через устройство при параллельном подключении в цепи.

    Добавить комментарий

    Ваш адрес email не будет опубликован. Обязательные поля помечены *